What is the scale factor of abc to def

What Is The Scale Factor Of Abc To Def

Answers

Answer 1

Answer:

[tex]\large \boxed{\mathrm{B. \ 6}}[/tex]

Step-by-step explanation:

Length AB is similar to length DE.

Scale factor = DE/AB

Scale factor = 12/2

Scale factor = 6

Answer 2

The required Scale factor would be 6 to transform triangle ΔABC to triangle ΔDEF which is the correct option (B).

What is a scale image?

Scale image is defined as a ratio that represents the relationship between the shape and size of a figure and the corresponding dimensions of the actual figure or object.

Length AB is similar to length DE.

Here triangle ΔABC is dilated to form triangle ΔDEF

Length of the side DE = 12

Length of the side AB = 2

As we know scale factor is the ratio of sides in the original image and the image after dilation.

Scale factor = DE/AB

Substitute the values of the length of the side DE and AB,

Scale factor = 12/2

Apply the division operation, and we get

Scale factor = 6

Therefore, the required Scale factor would be 6 to transform triangle ΔABC to triangle ΔDEF.

Learn more about the Scale images here:

brainly.com/question/13194929

#SPJ5


Related Questions

Suppose triangle TIP and triangle TOP are isosceles triangles. Also suppose that TI=5, PI=7, and PO=11. What are all the possible lengths TO? Enter the possible values, separated by commas.

Answers

Three Answers:  5, 7, 11

===========================================

Explanation:

Refer to the diagram below.

In order for triangle TOP to be isosceles, the missing side x must be either 5 or 7. This way we have exactly two sides that are the same length.

--------

If TP = 5, then the value of y could be either 5 or 11 to ensure that triangle TIP has exactly two sides the same length.

If TP = 7, then y = 7 or y = 11 for similar reasons.

--------

Therefore, the possible lengths for segment TO are 5, 7, and 11.

Answer:

7, 11

Step-by-step explanation:

its right- trust me-

Evaluate the following expression.
28 – 10 – 15 = 3 =
and this is the order of operations

Answers

Answer:

28 - 10 - 15 - 3

=> 18 - 15 - 3

=> 3 - 3

=> 0

Another way:

=> 28 - 10 - 15 - 3

=> 28 - 25 - 3

=> 28 - 28

=> 0

4x- 3y = -13 -x + 6y = -44

Answers

Answer:

3x and 9y and -31 is the correct answer if you are bloviating.

Step-by-step explanation:

bloviate the factors of x and y.

(x,y) =(-19/3 , 56/9)

3/v=2/4? i need help plz

Answers

Answer:

v = 6

Step-by-step explanation:

3/v = 2/4

We can use cross products to solve

2v = 3*4

2v = 12

Divide each side by 2

2v/2 = 12/2

v =6

Answer:

v=6

Step-by-step explanation:

If this is making them equal, then you should know that 2/4 is 1/2 , so you could just multiply 3 by both the 1 and the 2 in 1/2 and 1 times 3 is 3 which is what we have and 2 times 3 is 6 which is v.

2x/9 +x/3 = 13/6, solve for x

Answers

Answer:

x = 3 9/10

Step-by-step explanation:

2x/9 +x/3 = 13/6

Get a common denominator on the left side

2x/9 + x/3 *3/3 = 13/6

2x/9 + 3x/9 = 13/6

5x/9 = 13/6

Multiply each side by 9/5 to isolate x

5x/9 *9/5 = 13/6 * 9/5

x = 117/30

Divide the top and bottom by 3

x = 39/10

x = 3 9/10

Answer:

[tex]\bold{\red{\boxed{\blue{ x = 3.9}}}}[/tex]

Step-by-step explanation:

[tex] \frac{2x}{9} + \frac{x}{3} = \frac{13}{6} \\ \frac{2x + 3x}{9} = \frac{13}{6} \\ \frac{5x}{9} = \frac{13}{6} \\ use \: \: cross \: \: multipication \\ 5x \times 6 = 9 \times 13 \\ 30x =11 7 \\ \frac{30x}{30} = \frac{117}{30} \\ x = 3.9[/tex]

I REALLY need help with these 3 questions plz!!!!

Answers

Answer:

6. No. See explanation below.

7. 18 months

8. 16

Step-by-step explanation:

6. To rewrite a sum of two numbers using the distributive property, the two numbers must have a common factor greater than 1.

Let's find the GCF of 85 and 99:

85 = 5 * 17

99 = 3^2 + 11

5, 3, 11, and 17 are prime numbers. 85 and 99 have no prime factors in common. The GCF of 85 and 99 is 1, so the distributive property cannot be used on the sum 85 + 99.

Answer: No because the GCF of 85 and 99 is 1.

7.

We can solve this problem with the lest common multiple. We need to find a number of a month that is a multiple of both 6 and 9.

6 = 2 * 3

9 = 3^2

LCM = 2 * 3^2 = 2 * 9 = 18

Answer: 18 months

We can also answer this problem with a chart. We write the month number and whether they are home or on a trip. Then we look for the first month in which both are on a trip.

Month                Charlie          Dasha

1                          home             home

2                          home             home

3                          home             home

4                          home             home

5                          home             home

6                         trip                home

7                          home             home

8                          home             home

9                          home             trip

10                         home             home

11                         home             home

12                         trip               home

13                         home             home

14                         home             home

15                         home             home

16                         home             home

17                         home             home

18                         trip                 trip

Answer: 18 months

8.

First, we find the prime factorizations of 96 an 80.

96 = 2^5 * 3

80 = 2^4 *5

GCF = 2^4 = 16

Answer: 16

Nathan, a tutor, buys 5 calculators for $7.50 each at a store, planning to provide one to each of his clients. However, the next day, he discovers that the same calculators has gone on sale for $5.00 and also discovers that he will only have three tutoring clients instead of five. He returns the five calculators and purchases three calculators at the new sale price. He uses the following expression to determine the amount he should receive back from the store. (5 x $7.50) - (3 x $5.00) Which of the following expressions could Nathan have used. 5 ($7.50 - $5.00). $7.50 - $5.00. (5 x $7.50) - $5.00. (5 x $7.50) - $5.00. 3 ($7.50 - $5.00) +2 x $7.50

Answers

Answer: 5 (7.50-3 )

Step-by-step explanation:

Given: Previous price of calculator = $7.50

Number of client =5

Total price = (Number of calculators) x (Price for each calculator)

Total price of 5 calculators = (5 x $7.50)

New price of calculator = $5.00

Number of client =3

Total price of 3 calculators  = (3 x $5)

Price will receive = (Total price of 5 calculators) -(Total price of 3 calculators  )

= (5 x $7.50) -  (3 x $5)

= 5 (7.50-3 )

Required expression: 5 (7.50-3 )

SOMEBODY PLEASE HELP ME ON THIS ; DUE TODAY, i’ll mark u the brainliest

Answers

Answer: Angle Addition Postulate

Step-by-step explanation:

According to the angle addition postulate, the measure of an angle formed by two angles side by side is the sum of the measures of the two angles. It is used to evaluate the measure of an angle formed by two or more angles .

In the given picture, we have ∠MRO and ∠MRS on line SRO.

So, ∠SRO = ∠MRO +∠MRS  [By angle addition postulate]

So the postulate that justify the statement " ∠SRO = ∠MRO +∠MRS" is Angle Addition Postulate.

this is progression
i need to know C plsssss
thankssssssssssßsssssss

Answers

Answer:

[tex]l = 28[/tex]

Step-by-step explanation:

Given

[tex]S = \sum (2k - 3); k = 4\ to\ l[/tex]

Required

What is l when S = 725

This can be solved using Sum of n terms of an AP;

[tex]S_n = \frac{n}{2}(T_1 + T_n)[/tex]

Where

[tex]S_n = 725[/tex]

[tex]T_1 = first\ term[/tex]

To get T1; we substitute 4 for k in 2k - 3

[tex]T_1 = 2 * 4 - 3[/tex]

[tex]T_1 = 8 - 3[/tex]

[tex]T_1 = 5[/tex]

[tex]T_n = last\ term[/tex]

To get Tn; we substitute l for k in 2k - 3

[tex]T_n = 2 * l - 3[/tex]

[tex]T_n = 2l - 3[/tex]

n = the number of terms;

Since k = 4 to l, then

[tex]n = l - 4 +1[/tex]

[tex]n = l - 3[/tex]

Substitute these values in [tex]S_n = \frac{n}{2}(T_1 + T_n)[/tex]

[tex]725 = \frac{l-3}{2}(5 + 2l - 3)[/tex]

Collect Like Terms

[tex]725 = \frac{l-3}{2}(2l + 5- 3)[/tex]

[tex]725 = \frac{l-3}{2}(2l + 2)[/tex]

Open the bracket

[tex]725 = \frac{l-3}{2} * 2l + \frac{l-3}{2} * 2[/tex]

[tex]725 = (l-3) * l + (l-3)[/tex]

[tex]725 = l^2-3l + l-3[/tex]

[tex]725 = l^2-2l -3[/tex]

Subtract 725 from both sides

[tex]725 - 725 = l^2-2l -3 - 725[/tex]

[tex]l^2-2l -3 - 725 = 0[/tex]

[tex]l^2-2l - 728 = 0[/tex]

[tex]l^2 + 26l - 28l - 728 = 0[/tex]

[tex]l(l + 26) - 28(l + 26) = 0[/tex]

[tex](l - 28)(l + 26) = 0[/tex]

[tex]l - 28 = 0[/tex] or [tex]l + 26 = 0[/tex]

[tex]l = 28[/tex] or [tex]l = -26[/tex]

But l must be positive;

Hence, [tex]l = 28[/tex]

The slope of the line whose equation is x + y = 6 is: -1 1 6

Answers

Answer:

the slope of the line x+y = 6 is -1.

Step-by-step explanation:

slope = - coefficient of x

----------------------

coefficient of y

slope = -1 /1

slope = -1

Answer:

A: -1

Step-by-step explanation:

What is the formula for finding mean or average?​

Answers

Answer:

LOOK BELOW

Step-by-step explanation:

I would not call the explanation a formula

All you have to do to solve mean or average is add all of the numbers up and divide by the total amount of numbers

so for example

0,2,4,0,2,3,2,8,6  <-------- lets find the mean/average

0+2+4+2+3+2+8+6= 27/amount of numbers

amount of numbers=9

(count the zeros too!)

27/9=3

3 is the mean or average!!!

It takes a copy machine 6 minutes to complete a job. How many minutes are left on a job if 2/3 of the job is now complete?

Answers

2/3 of the job was completed in 4 minutes. the job left will be completed in 2 minutes

solve 3(11)× =3,993 for x​

Answers

Hi there! :)

Answer:

[tex]\huge\boxed{x = 3}[/tex]

Given the equation:

[tex]3(11)^{x} = 3993[/tex]

Divide both sides by 3:

[tex](11)^{x} = 1331[/tex]

Rewrite both sides of the equation with a base of 11.

[tex]1331 = 11^{3}[/tex], therefore:

[tex](11)^{x} = 11^{3}[/tex]

x = 3.

Answer:

121

Step-by-step explanation:

121 x 33 = 3993

A man drove 16 mi directly east from his home, made a left turn at an intersection, and then traveled 2 mi north to his place of work. If a road was made directly from his home to his place of work, what would its distance be to the nearest tenth of a mile?

Answers

Answer:

16. 1 miles

Step-by-step explanation:

Using Pythagorean Theorem,

a^2 + b^2 = c^2

Since the road that goes from his home to work directly is c^2...

Plug in the rest of the numbers

16^2 + 2^2 = c^2

256 + 4 = c^2

260 = c^2

The reverse square of 260 is

16. 1 miles

Please help me solve this. With Reasoning
6 2/5 - 4 2/3

Answers

Answer:

1 and 11/15.

Step-by-step explanation:

To solve, we can separate the integers from the fractions.

(6 - 4) + (2/5 - 2/3)

= 2 + (6/15 - 10/15)

= 2 + (-4/15)

= 1 + 15/15 - 4/15

= 1 + 11/15

= 1 and 11/15

Hope this helps!

4 (hx - 1) -3 (x +h) ≡ 5 (x + k)
Work out the value of h and k
H and k are integer constants

Answers

Answer:

        4hx - 8x - 3h - 4

k  =  ------------------------

                  5

            8x + 5k + 4

h  =  ------------------------

              4x - 3

Step-by-step explanation:

4 (hx - 1) - 3 (x + h) = 5 (x + k)

4hx - 4 - 3 (x + h) = 5 (x + k)

4hx - 4 - 3x - 3h = 5 (x + k)

4hx - 4 - 3x - 3h = 5x + 5k                   add 3h both sides

4hx - 4 - 3x - 3h + 3h = 5x + 5k + 3h   simplify

4hx - 4 - 3x = 5x + 5k + 3h                   add 4 both sides

4hx - 4 - 3x + 4 = 5x + 5k + 3h + 4       simplify

4hx - 3x = 5x + 5k + 3h + 4                  subtract 5x from both sides

4hx - 3x - 5x = 5x + 5k + 3h + 4 - 5x   simplify

4hx - 8x = 5k + 3h + 4                          

4hx - 8x - 3h - 4 = 5k  

      4hx - 8x - 3h - 4

k  =  ------------------------

                5

solving for h;

4hx - 3h = 8x + 5k + 4

h(4x - 3) = 8x + 5k + 4

          8x + 5k + 4

h  =  ------------------------

            4x - 3

The value of h and k are h = (8x + 5k + 4) / (4x - 3) and k = (4hx - 8x - 3h - 4) / 5 respectively

Given:

4 (hx - 1) -3 (x +h) ≡ 5 (x + k)

open parenthesis

4hx - 4 - 3x - 3h = 5x + 5k

4hx - 4 - 3x - 3h - 5x - 5k = 0

4hx - 8x - 3h - 5k - 4 = 0

For k

4hx - 8x - 3h - 4 = 5k

[tex]k = (4hx - 8x - 3h - 4) / 5[/tex]

For h

4hx - 8x - 3h - 5k - 4 = 0

4hx - 3h = 8x + 5k + 4

h(4x - 3) = 8x + 5k + 4

[tex]h = (8x + 5k + 4) / (4x - 3)[/tex]

Therefore, the value of h and k are h = (8x + 5k + 4) / (4x - 3) and k = (4hx - 8x - 3h - 4) / 5 respectively

Read more:

https://brainly.com/question/21406377

A ship travels a distance of 700 km. On the return trip it averages 10km/hr faster and 8 hours less, tp travel the 700km back. Determine how long the original part of the trip took in hours

Answers

Answer:

The total duration of the trip is 48 hours.

Step-by-step explanation:

Let suppose that ship travels at constant speed during its travel. Each stage is represented by the following kinematic equation:

[tex]v =\frac{\Delta s}{\Delta t}[/tex]

Where:

[tex]\Delta s[/tex] - Travelled distance, measured in kilometers.

[tex]\Delta t[/tex] - Time, measured in hours.

[tex]v[/tex] - Speed, measured in kilometers per hour.

Now, each stage is represented by the following expressions:

Outbound trip

[tex]v = \frac{700\,km}{\Delta t}[/tex]

Return trip

[tex]v + 10\,\frac{km}{h} = \frac{700\,kh}{\Delta t - 8\,h}[/tex]

By eliminating [tex]v[/tex] and simplifying the resulting expression algebraically:

[tex]\frac{700\,km}{\Delta t} + 10\,\frac{km}{h} = \frac{700\,km}{\Delta t -8\,h}[/tex]

[tex](700\,km)\cdot \left(\frac{1}{\Delta t - 8\,h}-\frac{1}{\Delta t} \right) = 10\,\frac{km}{h}[/tex]

[tex]\frac{1}{\Delta t - 8\,h}-\frac{1}{\Delta t} = \frac{1}{70}\,\frac{1}{h}[/tex]

[tex]\frac{8\,h}{\Delta t \cdot (\Delta t-8\,h)} = \frac{1}{70}\,\frac{1}{h}[/tex]

[tex]560\,h^{2} = \Delta t\cdot (\Delta t - 8\,h)[/tex]

[tex](\Delta t )^{2}-8\cdot \Delta t - 560 = 0[/tex]

This equation can be solved by means of the Quadratic Formula, whose roots are presented below:

[tex]\Delta t_{1} = 28\,h[/tex] and [tex]\Delta t_{2} = -20\,h[/tex]

Only the first roots offers a physically resonable solution. Then, total duration of the trip is:

[tex]t_{T} = 28\,h +20\,h[/tex]

[tex]t_{T} = 48\,h[/tex]

The total duration of the trip is 48 hours.

For the mathematics projects, a teacher divides 27 students into 2 groups. One group has more students than twice the number of students in the other group by 3. Find the number of students in both groups.

Write as a equation.​

Answers

Answer:

8, 19

Step-by-step explanation:

let group 1 have x students and group 2 have y students

x + y = 27

but group 2 has 2x + 3 students

the sum of students from both groups is 27

x + 2x + 3 = 27

3x + 3 = 27

3x = 24

x = 8

y = 2x + 3

y = 19

PLEASE HELP MEEE How can a company use a scatter plot to make future sale decisions

Answers

Answer:

by tracking data of how much money was made on one product in a certain amount of time

Step-by-step explanation:

Find the vertex of the parabola.
f (x) = x squared minus 6 x + 13
a.
( 4, 0)
c.
( 3, 4)
b.
(0, 3)
d.
( 4, 3)

Answers

Answer:

The vertex is (3,4)

Step-by-step explanation:

f (x) = x^2 - 6 x + 13

Completing the square

-6/2 = -3 and squaring it = 9

    = x^2 -6x +9 +4

   = ( x-3) ^2 +4

The equation is now in vertex form

a( x-h) ^2 +k

where the vertex is ( h,k)

The vertex is (3,4)

Answer:

C on edge

Step-by-step explanation:

2x-15=3(2x+3)
It’s multi step equations

Answers

x=−6 quick maths xd xd xd

Answer:

x = -6

Step-by-step explanation:

Hello!

2x - 15 = 3(2x + 3)

Distribute the 3

3 * 2x = 6x

3 * 3 = 9

2x - 15 = 6x + 9

Subtract 2x from both sides

-15 = 4x + 9

Subtract 9 from both sides

-24 = 4x

Divide both sides by 4

-6 = x

Hope this helps!

Pls answer this question as soon as possible​

Answers

Answer:

The answer is -½.

✌ yeah it is ✌

a 6 foot tall man casts a shadow that is 9 ft long. At the same time, a tree nearby casts a 48 ft shadow. how tall is the tree

Answers

Answer:

32 ft tall

Step-by-step explanation:

Since a 6 ft man casts a shadow 9 ft long, the shadow is 3/2 of the actual object/person.

SINCE THE TREE'S SHADOW IS AT THE SAME TIME, THE HEIGHT IS THE SAME RULE.

We know the tree's shadow is 48 ft.

--> 48/3 = 16

16 x 2 = 32

32 ft tall

Hope this helps!

Answer: 32ft tall

Step-by-step explanation:

PLEASE HELP!!!!!! URGENT!!!!!!!!! ALGEBRA! Hannah, Yu, and Becky went to buy groceries together. All three of them bought pasta, noodles, and split peas. Hannah bought 3 packs of pasta, 10 packs of noodles, and 2 packs of split peas. Yu bought 5 packs of pasta, 12 packs of noodles, and 1 pack of split peas. Becky bought 3 packs of pasta, 15 packs of noodles, and 2 packs of split peas. Hannah, Yu, and Becky spent $8.41, $9.42, and $9.66 respectively on these items. What are the costs of a pack of pasta, a pack of noodles, and a pack of split peas? A) Pasta: $0.69 per pack, Noodles: $0.25 per pack, Split peas: $0.99 per pack B) Pasta: $1.09 per pack, Noodles: $0.20 per pack, Split peas: $1.97 per pack C) Pasta: $0.99 per pack, Noodles: $0.25 per pack, Split peas: $1.47 per pack D) Pasta: $0.69 per pack, Noodles: $0.20 per pack, Split peas: $1.97 per pack

Answers

Answer:

option d

sorry it took so long.

Simplify the following expression. (10-4i)(4-5i)+(-15+20i)

Answers

Answer:

5-46i

Step-by-step explanation:

1. Multiply (10-4i) and (4-5i), I recomnd using foil:

40-50i-16+20i^2 + (-15+20i)

2. Remove the parenthesis around -15+20i

*we can do this since there is a "+":    

40-50i-16+20i^2 + (-15)+20

3. Simplify i^2

* i^2 is -1 by textbook defination:

40-50i-16+20(-1) + (-15)+20

4. Simplify

40-50i-16-20 + (-15)+20

6. Combine like terms:

-5-50i-16i+20i

5-46i

And the problem is done

Help me with 1 please

Answers

Answer:

[tex]\huge\boxed{Mass = 11600\ kg}[/tex]

Step-by-step explanation:

Given:

Density = ρ = 2900 kg/m³

Volume = V = 4 m³

Required:

Mass = m = ?

Formula:

Mass = Density × Volume

Solution:

Mass = 2900 * 4

Mass = 11600 kg

heyy
the answer is

mass=11600 kg

Simplify cos^2theta(1+ tan^2theta)

Answers

Answer:

1

Step-by-step explanation:

We will use x instead of theta

● cos^2 x *(1+tan^2x)

We khow that: 1+ tan^2 x = 1/cos^2 x

Replace 1+tan^2 x by the new expression

● cos^2 x (1/cos^2 x)

● cos^2x/ cos^2 x

● 1

What is the value of the expression below when x = 5 and y = 2?
X + 6y​

Answers

instead of X put 5 and instead of y put 2
5+6(2)
5+12= 17

The value of the expression x + 6y when x = 5 and y = 2 is 17.

What is an expression?

An expression is a way of writing a statement with more than two variables or numbers with operations such as addition, subtraction, multiplication, and division.

Example: 2 + 3x + 4y = 7 is an expression.

We have,

x + 6y

When x = 5 and y = 2

= x + 6y

= 5 + 6 x 2

= 5 + 12

= 17

Thus,

The value is 17.

Learn more about expressions here:

https://brainly.com/question/3118662

#SPJ2

Simplify -1-7 +41. N

Answers

Answer:

-3

Step-by-step explanation:

-7+4=-3

The absolute value of -3 is 3

The negative sign in front of the absolute value bracket makes it -3

Olivia has 4 2/3 yards of fabric to make scarves. She needs 3/4 yards for one scarf. How many
scarves can she make?​

Answers

Answer:

6 scarves

Step-by-step explanation:

So we know that 3/4 yd. = 1 (scarf)

We have 4 2/3 material to make the scarves

=> convert to an improper fraction 4 2/3 = 14/3

=> Divide material by needed amt.

=> 14/3 / 3/4 = 14/3 x 4/3

=> 14/3 x 4/3 = 56/9

56/9 = 6 2/9

But 6 2/9 is not our answer. Since we need a full amt. of scraves, we round down to our final answer of 6 scarves.

Hope this helps!

Other Questions
For how many years are primary teeth needed for chewing, appearance, and speech? what fears were raised during the early period of federal discussion in Nepal What is an important aspect for toddlers that parents can provide? A safe environment to explore Plenty of toys for entertainment Punishment when they dont behave correctly All of the above PLEASE HELP I WILL GIVE BRAINLIEST Complete the frequency table: Method of Travel to School Walk/Bike Bus Car Row totals Under age 15 60 165 Age 15 and above 65 195 Column totals 152 110 98 360 What percentage of students under age 15 travel to school by car? Round to the nearest whole percent. 11% 18% 41% 80% A broker arrives to present a purchase offer to a disabled woman and finds her son and his wife also present. In the presence of the broker, both individuals persistently urge the woman to accept the offer, even though it is much lower than the price she has been asking for her home. If the woman accepts the offer, she may later claim that If f(x) = 32 +1 and g(x) = 1-, what is the value of (f-g)(2)?12143638 what is puberty explain it Marco purchased a large box of comic books for $300. He gave 15 of the comic books to his brother and then sold the rest on an internet website for $330 making a profit , making a profit of $1.50 on each one.how many comic books were in the box? what was the original price of each comic book (assuming they all cost the same amount)? In the multiplication below, each of A, B andC represents a different digit. What is ABC?A B CX3 Which of the following elements has a complete outer shell of electrons? A. Iron (Fe) B. Hydrogen (H) C. Neon (Ne) D. Nitrogen (N) A group of air particles in a balloon are spread far apart and moving rapidly around inside the balloon. The balloon is placed in a refrigerator for 30 minutes. What would you expect to happen to the appearance of the balloon? Explain your answer in terms of particle movement and their change in temperature. The balloon is removed from the refrigerator and placed on the kitchen counter. What would you expect to happen to the appearance of the balloon? Explain your answer in terms of particle movement and their change in temperature. Question 59 of 83 Project M requires an initial investment of $25 million. The project is expected to generate $2.25 million in after-tax cash flow each year forever. Calculate the IRR for the project. 10% 9% 8% 7% Discount factor is 0.985. Stock XYZ is selling for $40 a share. An American option on this stock with a strike price of $38 is trading at $0.25 per share. If it is known that this option is priced above its intrinsic value, what type of option is it? Given the gene sequence GGACCGTCGATCTTC, which of the following choices would represent an inversion mutation? A. GGACTCGATCTTC B. CCACCGTCGATCTTC C. GGACCGTCGATCTTC D. GGACCGTCGATCCTT look at picture and solve Write a short dialogueboth sides of the conversationin Spanish between you and your new Spanish instructor in complete sentences. Use the vocabulary from this lesson and the following suggestions as a guide for your answer: You may copy and paste the accented and special characters from this list if needed: , , , , , , , , , , , , , , *Note: The sample sentences/questions in parentheses are just a guide to help you form your sentences/questions. You must come up with your own original answers keeping academic integrity intact. Greet and state your name in one sentence. (e.g., Good morning, my name is ___.) Ask your instructor his or her name in one sentence. (e.g., What is your name?formal) Have your instructor reply in one sentence what his/her name is. (e.g., My name is ___.) Let your instructor know you're glad to meet him or her in one sentence/expression. (e.g., Nice to meet you, sir.) Give a farewell. (e.g., Goodbye.) NASA is giving serious consideration to the concept of solar sailing. A solar sailcraft uses a large, low- mass sail and the energy and momentum of sunlight for propulsion.Should the sail be absorbing or reflective? Why? a. The sail should be reflective because in this case the momentum transferred to the sail per unit area per unit time is smaller than for absorbing sail, therefore the radiation pressure is larger for the reflective sail b. The sail should be absorbing because in this case the momentum transferred to the sail per unit area per unit time is larger than for reflective sail, therefore the radiation pressure is larger for the absorbing sail c. The sail should be absorbing because in this case the momentum transferred to the sail per unit area per unit time is smaller than for reflective sail, therefore the radiation pressure is larger for the absorbing sail. d. The sail should be reflective because in this case the momentum transferred to the sail per unit area per unit time is larger than for absorbing sail, therefore the radiation pressure is larger for the reflective sail plsss help meeeeeee!!!! Jakes club has 35 members. it's rules require that 60% of them must be present for any vote. at least how many members must be present to have a vote? 550 J of heat is added to the gas in an isothermal process. As the gas expands, pushing against the piston, how much work does it do Identify the domain of the function shown in the graph.A. x is all numbers.B. X20O C. 0D. x